Prove $sum_kmid nmu(k)d(k)=(-1)^omega(n)$ The Next CEO of Stack OverflowShowing $sum_dmid n mu(d)tau(n/d)=1$ and $sum_dmid n mu(d)tau(d)=(-1)^r$Möbius function verificationProve $sum_k = 1^n mu(k)left[ frac nk right] = 1$Convolution identity involving the Möbius function $sum_n,d>0 |mu(d)| = 2^omega(n)$Prove that $sum_t vert n d^3(t) = (sum_t vert nd(t))^2$ for all $n in mathbbN$Proof of inequality involving multiplicative function?Bound for the sum of the divisors of a numberProve that $sum_n, d geq 1 = 2^omega(n)$Formula for unique distribution of colored balls into boxesUpper bound for the divisor counting function?How to invert an arithmetic function where Möbius inversion may not apply?

Fastest way to shutdown Ubuntu Mate 18.10

I believe this to be a fraud - hired, then asked to cash check and send cash as Bitcoin

How do I go from 300 unfinished/half written blog posts, to published posts?

What makes a siege story/plot interesting?

What is the difference between "behavior" and "behaviour"?

When airplanes disconnect from a tanker during air to air refueling, why do they bank so sharply to the right?

How did people program for Consoles with multiple CPUs?

What is the purpose of the Evocation wizard's Potent Cantrip feature?

Text adventure game code

Why were Madagascar and New Zealand discovered so late?

How to Reset Passwords on Multiple Websites Easily?

What's the point of interval inversion?

What do "high sea" and "carry" mean in this sentence?

Whats the best way to handle refactoring a big file?

Robert Sheckley short story about vacation spots being overwhelmed

If I blow insulation everywhere in my attic except the door trap, will heat escape through it?

How should I support this large drywall patch?

Is it okay to store user locations?

Are there languages with no euphemisms?

MAZDA 3 2006 (UK) - poor acceleration then takes off at 3250 revs

Inappropriate reference requests from Journal reviewers

How long to clear the 'suck zone' of a turbofan after start is initiated?

Science fiction (dystopian) short story set after WWIII

How do I construct this japanese bowl?



Prove $sum_kmid nmu(k)d(k)=(-1)^omega(n)$



The Next CEO of Stack OverflowShowing $sum_dmid n mu(d)tau(n/d)=1$ and $sum_dmid n mu(d)tau(d)=(-1)^r$Möbius function verificationProve $sum_k = 1^n mu(k)left[ frac nk right] = 1$Convolution identity involving the Möbius function $sum_n,d>0 |mu(d)| = 2^omega(n)$Prove that $sum_t vert n d^3(t) = (sum_t vert nd(t))^2$ for all $n in mathbbN$Proof of inequality involving multiplicative function?Bound for the sum of the divisors of a numberProve that $sum_n, d geq 1 = 2^omega(n)$Formula for unique distribution of colored balls into boxesUpper bound for the divisor counting function?How to invert an arithmetic function where Möbius inversion may not apply?










5












$begingroup$


I have the following exercise.




Show that for all natural numbers $n$, the following equality holds
$$sum_dmu(d)d(d)=(-1)^omega(n)$$
Here, $mu$ is the Möbius function, $d$ counts the number of divisors of $n$, and $omega$ counts the number of distinct prime divisors of $n$.




I tried looking at a number like $n=10$ just to see what it looks like expanded. So since the divisors of $n$ are $1,2,5,$ and $10$, I can show that
$$sum_10mu(d)d(d)=(-1)^omega(10)=mu(1)d(1)+mu(2)d(2)+mu(5)d(5)+mu(10)d(10)=1$$
This gives me
$$1cdot 1+-1cdot 2+-1cdot 2 +1cdot 4 $$
I'm thinking somehow since both $mu$ and $d$ are multiplicative that we can rewrite though as
$$mu(1)d(1)+mu(2)d(2)+mu(5)d(5)+mu(2)d(2)mu(5)d(5)$$
$$=mu(1)d(1)+mu(5)d(5)+mu(2)d(2)+mu(2)d(2)mu(5)d(5)$$
$$=mu(1)d(1)+mu(5)d(5)+mu(2)d(2)(1+mu(5)d(5))$$
$$=mu(1)d(1)+mu(5)d(5)+mu(2)d(2)(mu(1)d(1)+mu(5)d(5))$$
$$=(1+mu(2)d(2))(1+mu(5)d(5))$$
$$=sum_dmu(d)d(d)sum_dmu(d)d(d)$$
So this original summation function is multiplicative. But this isn't helping me see the how to move forward. I know that the $mu$ function is defined as $mu(n)=(-1)^omega(n)$ if $n$ is square free and $0$ if divisible by a square, so I think this plays a role somehow, but again, I'm feeling lost.



EDIT: Would looking at $n=prod_i=1^kp_i^alpha_i$ be a more useful approach to the problem? Knowing the larger summed function is multiplicative means I can focus my approach on the $p_i^alpha_i$...










share|cite|improve this question











$endgroup$











  • $begingroup$
    The book I am working out of is Niven's Introduction to the Theory of Numbers and they use $d$, so I've just gotten in the habit of using it. I know it's confusing. My professor did reveal that $tau$ was also used, but I guess I'm just used to the idea of $d$=divisor function.
    $endgroup$
    – Lalaloopsy
    Jul 19 '14 at 18:32






  • 5




    $begingroup$
    $d(d)$ is such a confusing notation...
    $endgroup$
    – CuriousGuest
    Jul 20 '14 at 6:54















5












$begingroup$


I have the following exercise.




Show that for all natural numbers $n$, the following equality holds
$$sum_dmu(d)d(d)=(-1)^omega(n)$$
Here, $mu$ is the Möbius function, $d$ counts the number of divisors of $n$, and $omega$ counts the number of distinct prime divisors of $n$.




I tried looking at a number like $n=10$ just to see what it looks like expanded. So since the divisors of $n$ are $1,2,5,$ and $10$, I can show that
$$sum_10mu(d)d(d)=(-1)^omega(10)=mu(1)d(1)+mu(2)d(2)+mu(5)d(5)+mu(10)d(10)=1$$
This gives me
$$1cdot 1+-1cdot 2+-1cdot 2 +1cdot 4 $$
I'm thinking somehow since both $mu$ and $d$ are multiplicative that we can rewrite though as
$$mu(1)d(1)+mu(2)d(2)+mu(5)d(5)+mu(2)d(2)mu(5)d(5)$$
$$=mu(1)d(1)+mu(5)d(5)+mu(2)d(2)+mu(2)d(2)mu(5)d(5)$$
$$=mu(1)d(1)+mu(5)d(5)+mu(2)d(2)(1+mu(5)d(5))$$
$$=mu(1)d(1)+mu(5)d(5)+mu(2)d(2)(mu(1)d(1)+mu(5)d(5))$$
$$=(1+mu(2)d(2))(1+mu(5)d(5))$$
$$=sum_dmu(d)d(d)sum_dmu(d)d(d)$$
So this original summation function is multiplicative. But this isn't helping me see the how to move forward. I know that the $mu$ function is defined as $mu(n)=(-1)^omega(n)$ if $n$ is square free and $0$ if divisible by a square, so I think this plays a role somehow, but again, I'm feeling lost.



EDIT: Would looking at $n=prod_i=1^kp_i^alpha_i$ be a more useful approach to the problem? Knowing the larger summed function is multiplicative means I can focus my approach on the $p_i^alpha_i$...










share|cite|improve this question











$endgroup$











  • $begingroup$
    The book I am working out of is Niven's Introduction to the Theory of Numbers and they use $d$, so I've just gotten in the habit of using it. I know it's confusing. My professor did reveal that $tau$ was also used, but I guess I'm just used to the idea of $d$=divisor function.
    $endgroup$
    – Lalaloopsy
    Jul 19 '14 at 18:32






  • 5




    $begingroup$
    $d(d)$ is such a confusing notation...
    $endgroup$
    – CuriousGuest
    Jul 20 '14 at 6:54













5












5








5


2



$begingroup$


I have the following exercise.




Show that for all natural numbers $n$, the following equality holds
$$sum_dmu(d)d(d)=(-1)^omega(n)$$
Here, $mu$ is the Möbius function, $d$ counts the number of divisors of $n$, and $omega$ counts the number of distinct prime divisors of $n$.




I tried looking at a number like $n=10$ just to see what it looks like expanded. So since the divisors of $n$ are $1,2,5,$ and $10$, I can show that
$$sum_10mu(d)d(d)=(-1)^omega(10)=mu(1)d(1)+mu(2)d(2)+mu(5)d(5)+mu(10)d(10)=1$$
This gives me
$$1cdot 1+-1cdot 2+-1cdot 2 +1cdot 4 $$
I'm thinking somehow since both $mu$ and $d$ are multiplicative that we can rewrite though as
$$mu(1)d(1)+mu(2)d(2)+mu(5)d(5)+mu(2)d(2)mu(5)d(5)$$
$$=mu(1)d(1)+mu(5)d(5)+mu(2)d(2)+mu(2)d(2)mu(5)d(5)$$
$$=mu(1)d(1)+mu(5)d(5)+mu(2)d(2)(1+mu(5)d(5))$$
$$=mu(1)d(1)+mu(5)d(5)+mu(2)d(2)(mu(1)d(1)+mu(5)d(5))$$
$$=(1+mu(2)d(2))(1+mu(5)d(5))$$
$$=sum_dmu(d)d(d)sum_dmu(d)d(d)$$
So this original summation function is multiplicative. But this isn't helping me see the how to move forward. I know that the $mu$ function is defined as $mu(n)=(-1)^omega(n)$ if $n$ is square free and $0$ if divisible by a square, so I think this plays a role somehow, but again, I'm feeling lost.



EDIT: Would looking at $n=prod_i=1^kp_i^alpha_i$ be a more useful approach to the problem? Knowing the larger summed function is multiplicative means I can focus my approach on the $p_i^alpha_i$...










share|cite|improve this question











$endgroup$




I have the following exercise.




Show that for all natural numbers $n$, the following equality holds
$$sum_dmu(d)d(d)=(-1)^omega(n)$$
Here, $mu$ is the Möbius function, $d$ counts the number of divisors of $n$, and $omega$ counts the number of distinct prime divisors of $n$.




I tried looking at a number like $n=10$ just to see what it looks like expanded. So since the divisors of $n$ are $1,2,5,$ and $10$, I can show that
$$sum_10mu(d)d(d)=(-1)^omega(10)=mu(1)d(1)+mu(2)d(2)+mu(5)d(5)+mu(10)d(10)=1$$
This gives me
$$1cdot 1+-1cdot 2+-1cdot 2 +1cdot 4 $$
I'm thinking somehow since both $mu$ and $d$ are multiplicative that we can rewrite though as
$$mu(1)d(1)+mu(2)d(2)+mu(5)d(5)+mu(2)d(2)mu(5)d(5)$$
$$=mu(1)d(1)+mu(5)d(5)+mu(2)d(2)+mu(2)d(2)mu(5)d(5)$$
$$=mu(1)d(1)+mu(5)d(5)+mu(2)d(2)(1+mu(5)d(5))$$
$$=mu(1)d(1)+mu(5)d(5)+mu(2)d(2)(mu(1)d(1)+mu(5)d(5))$$
$$=(1+mu(2)d(2))(1+mu(5)d(5))$$
$$=sum_dmu(d)d(d)sum_dmu(d)d(d)$$
So this original summation function is multiplicative. But this isn't helping me see the how to move forward. I know that the $mu$ function is defined as $mu(n)=(-1)^omega(n)$ if $n$ is square free and $0$ if divisible by a square, so I think this plays a role somehow, but again, I'm feeling lost.



EDIT: Would looking at $n=prod_i=1^kp_i^alpha_i$ be a more useful approach to the problem? Knowing the larger summed function is multiplicative means I can focus my approach on the $p_i^alpha_i$...







number-theory multiplicative-function divisor-counting-function mobius-inversion dirichlet-convolution






share|cite|improve this question















share|cite|improve this question













share|cite|improve this question




share|cite|improve this question








edited yesterday









Eric Wofsey

191k14216349




191k14216349










asked Jul 19 '14 at 18:19









LalaloopsyLalaloopsy

89111221




89111221











  • $begingroup$
    The book I am working out of is Niven's Introduction to the Theory of Numbers and they use $d$, so I've just gotten in the habit of using it. I know it's confusing. My professor did reveal that $tau$ was also used, but I guess I'm just used to the idea of $d$=divisor function.
    $endgroup$
    – Lalaloopsy
    Jul 19 '14 at 18:32






  • 5




    $begingroup$
    $d(d)$ is such a confusing notation...
    $endgroup$
    – CuriousGuest
    Jul 20 '14 at 6:54
















  • $begingroup$
    The book I am working out of is Niven's Introduction to the Theory of Numbers and they use $d$, so I've just gotten in the habit of using it. I know it's confusing. My professor did reveal that $tau$ was also used, but I guess I'm just used to the idea of $d$=divisor function.
    $endgroup$
    – Lalaloopsy
    Jul 19 '14 at 18:32






  • 5




    $begingroup$
    $d(d)$ is such a confusing notation...
    $endgroup$
    – CuriousGuest
    Jul 20 '14 at 6:54















$begingroup$
The book I am working out of is Niven's Introduction to the Theory of Numbers and they use $d$, so I've just gotten in the habit of using it. I know it's confusing. My professor did reveal that $tau$ was also used, but I guess I'm just used to the idea of $d$=divisor function.
$endgroup$
– Lalaloopsy
Jul 19 '14 at 18:32




$begingroup$
The book I am working out of is Niven's Introduction to the Theory of Numbers and they use $d$, so I've just gotten in the habit of using it. I know it's confusing. My professor did reveal that $tau$ was also used, but I guess I'm just used to the idea of $d$=divisor function.
$endgroup$
– Lalaloopsy
Jul 19 '14 at 18:32




5




5




$begingroup$
$d(d)$ is such a confusing notation...
$endgroup$
– CuriousGuest
Jul 20 '14 at 6:54




$begingroup$
$d(d)$ is such a confusing notation...
$endgroup$
– CuriousGuest
Jul 20 '14 at 6:54










2 Answers
2






active

oldest

votes


















7












$begingroup$

Look at the sum for a prime power $p^k$. The divisors of $p^k$ are $1,p,p^2,...,p^k-1$. All of them contain a square except $1$ and $p$. That means $mu(p^a)=0$. So the sum is
$$mu(1)d(1)+mu(p)d(p)\=1times 1+(-1)times 2=1-2=-1$$
So each different prime, or its power, contributes a factor (-1).






share|cite|improve this answer









$endgroup$








  • 1




    $begingroup$
    In particular, since $mu(n)d(n)$ is a multiplicative function, so is $sum_n'mid n mu(n')d(n')$...
    $endgroup$
    – Thomas Andrews
    Jul 19 '14 at 18:32


















5












$begingroup$

Another, Combinatorial, way would be like $$sum _d mu (d) d(d)=sum _i=0^w(n)sum _p_1<p_2 ldots < p_imu (p_1 dots p_i)d (p_1 dots p_i)=sum _i=0^w(n)binomw(n)i(-1)^i2^i=(1-2)^w(n)$$ where the $p_j$ are primes in the descomposition of $n$.






share|cite|improve this answer









$endgroup$













    Your Answer





    StackExchange.ifUsing("editor", function ()
    return StackExchange.using("mathjaxEditing", function ()
    StackExchange.MarkdownEditor.creationCallbacks.add(function (editor, postfix)
    StackExchange.mathjaxEditing.prepareWmdForMathJax(editor, postfix, [["$", "$"], ["\\(","\\)"]]);
    );
    );
    , "mathjax-editing");

    StackExchange.ready(function()
    var channelOptions =
    tags: "".split(" "),
    id: "69"
    ;
    initTagRenderer("".split(" "), "".split(" "), channelOptions);

    StackExchange.using("externalEditor", function()
    // Have to fire editor after snippets, if snippets enabled
    if (StackExchange.settings.snippets.snippetsEnabled)
    StackExchange.using("snippets", function()
    createEditor();
    );

    else
    createEditor();

    );

    function createEditor()
    StackExchange.prepareEditor(
    heartbeatType: 'answer',
    autoActivateHeartbeat: false,
    convertImagesToLinks: true,
    noModals: true,
    showLowRepImageUploadWarning: true,
    reputationToPostImages: 10,
    bindNavPrevention: true,
    postfix: "",
    imageUploader:
    brandingHtml: "Powered by u003ca class="icon-imgur-white" href="https://imgur.com/"u003eu003c/au003e",
    contentPolicyHtml: "User contributions licensed under u003ca href="https://creativecommons.org/licenses/by-sa/3.0/"u003ecc by-sa 3.0 with attribution requiredu003c/au003e u003ca href="https://stackoverflow.com/legal/content-policy"u003e(content policy)u003c/au003e",
    allowUrls: true
    ,
    noCode: true, onDemand: true,
    discardSelector: ".discard-answer"
    ,immediatelyShowMarkdownHelp:true
    );



    );













    draft saved

    draft discarded


















    StackExchange.ready(
    function ()
    StackExchange.openid.initPostLogin('.new-post-login', 'https%3a%2f%2fmath.stackexchange.com%2fquestions%2f871933%2fprove-sum-k-mid-n-mukdk-1-omegan%23new-answer', 'question_page');

    );

    Post as a guest















    Required, but never shown

























    2 Answers
    2






    active

    oldest

    votes








    2 Answers
    2






    active

    oldest

    votes









    active

    oldest

    votes






    active

    oldest

    votes









    7












    $begingroup$

    Look at the sum for a prime power $p^k$. The divisors of $p^k$ are $1,p,p^2,...,p^k-1$. All of them contain a square except $1$ and $p$. That means $mu(p^a)=0$. So the sum is
    $$mu(1)d(1)+mu(p)d(p)\=1times 1+(-1)times 2=1-2=-1$$
    So each different prime, or its power, contributes a factor (-1).






    share|cite|improve this answer









    $endgroup$








    • 1




      $begingroup$
      In particular, since $mu(n)d(n)$ is a multiplicative function, so is $sum_n'mid n mu(n')d(n')$...
      $endgroup$
      – Thomas Andrews
      Jul 19 '14 at 18:32















    7












    $begingroup$

    Look at the sum for a prime power $p^k$. The divisors of $p^k$ are $1,p,p^2,...,p^k-1$. All of them contain a square except $1$ and $p$. That means $mu(p^a)=0$. So the sum is
    $$mu(1)d(1)+mu(p)d(p)\=1times 1+(-1)times 2=1-2=-1$$
    So each different prime, or its power, contributes a factor (-1).






    share|cite|improve this answer









    $endgroup$








    • 1




      $begingroup$
      In particular, since $mu(n)d(n)$ is a multiplicative function, so is $sum_n'mid n mu(n')d(n')$...
      $endgroup$
      – Thomas Andrews
      Jul 19 '14 at 18:32













    7












    7








    7





    $begingroup$

    Look at the sum for a prime power $p^k$. The divisors of $p^k$ are $1,p,p^2,...,p^k-1$. All of them contain a square except $1$ and $p$. That means $mu(p^a)=0$. So the sum is
    $$mu(1)d(1)+mu(p)d(p)\=1times 1+(-1)times 2=1-2=-1$$
    So each different prime, or its power, contributes a factor (-1).






    share|cite|improve this answer









    $endgroup$



    Look at the sum for a prime power $p^k$. The divisors of $p^k$ are $1,p,p^2,...,p^k-1$. All of them contain a square except $1$ and $p$. That means $mu(p^a)=0$. So the sum is
    $$mu(1)d(1)+mu(p)d(p)\=1times 1+(-1)times 2=1-2=-1$$
    So each different prime, or its power, contributes a factor (-1).







    share|cite|improve this answer












    share|cite|improve this answer



    share|cite|improve this answer










    answered Jul 19 '14 at 18:26









    Empy2Empy2

    33.6k12462




    33.6k12462







    • 1




      $begingroup$
      In particular, since $mu(n)d(n)$ is a multiplicative function, so is $sum_n'mid n mu(n')d(n')$...
      $endgroup$
      – Thomas Andrews
      Jul 19 '14 at 18:32












    • 1




      $begingroup$
      In particular, since $mu(n)d(n)$ is a multiplicative function, so is $sum_n'mid n mu(n')d(n')$...
      $endgroup$
      – Thomas Andrews
      Jul 19 '14 at 18:32







    1




    1




    $begingroup$
    In particular, since $mu(n)d(n)$ is a multiplicative function, so is $sum_n'mid n mu(n')d(n')$...
    $endgroup$
    – Thomas Andrews
    Jul 19 '14 at 18:32




    $begingroup$
    In particular, since $mu(n)d(n)$ is a multiplicative function, so is $sum_n'mid n mu(n')d(n')$...
    $endgroup$
    – Thomas Andrews
    Jul 19 '14 at 18:32











    5












    $begingroup$

    Another, Combinatorial, way would be like $$sum _d mu (d) d(d)=sum _i=0^w(n)sum _p_1<p_2 ldots < p_imu (p_1 dots p_i)d (p_1 dots p_i)=sum _i=0^w(n)binomw(n)i(-1)^i2^i=(1-2)^w(n)$$ where the $p_j$ are primes in the descomposition of $n$.






    share|cite|improve this answer









    $endgroup$

















      5












      $begingroup$

      Another, Combinatorial, way would be like $$sum _d mu (d) d(d)=sum _i=0^w(n)sum _p_1<p_2 ldots < p_imu (p_1 dots p_i)d (p_1 dots p_i)=sum _i=0^w(n)binomw(n)i(-1)^i2^i=(1-2)^w(n)$$ where the $p_j$ are primes in the descomposition of $n$.






      share|cite|improve this answer









      $endgroup$















        5












        5








        5





        $begingroup$

        Another, Combinatorial, way would be like $$sum _d mu (d) d(d)=sum _i=0^w(n)sum _p_1<p_2 ldots < p_imu (p_1 dots p_i)d (p_1 dots p_i)=sum _i=0^w(n)binomw(n)i(-1)^i2^i=(1-2)^w(n)$$ where the $p_j$ are primes in the descomposition of $n$.






        share|cite|improve this answer









        $endgroup$



        Another, Combinatorial, way would be like $$sum _d mu (d) d(d)=sum _i=0^w(n)sum _p_1<p_2 ldots < p_imu (p_1 dots p_i)d (p_1 dots p_i)=sum _i=0^w(n)binomw(n)i(-1)^i2^i=(1-2)^w(n)$$ where the $p_j$ are primes in the descomposition of $n$.







        share|cite|improve this answer












        share|cite|improve this answer



        share|cite|improve this answer










        answered Jul 21 '14 at 2:23









        PhicarPhicar

        2,6601915




        2,6601915



























            draft saved

            draft discarded
















































            Thanks for contributing an answer to Mathematics Stack Exchange!


            • Please be sure to answer the question. Provide details and share your research!

            But avoid


            • Asking for help, clarification, or responding to other answers.

            • Making statements based on opinion; back them up with references or personal experience.

            Use MathJax to format equations. MathJax reference.


            To learn more, see our tips on writing great answers.




            draft saved


            draft discarded














            StackExchange.ready(
            function ()
            StackExchange.openid.initPostLogin('.new-post-login', 'https%3a%2f%2fmath.stackexchange.com%2fquestions%2f871933%2fprove-sum-k-mid-n-mukdk-1-omegan%23new-answer', 'question_page');

            );

            Post as a guest















            Required, but never shown





















































            Required, but never shown














            Required, but never shown












            Required, but never shown







            Required, but never shown

































            Required, but never shown














            Required, but never shown












            Required, but never shown







            Required, but never shown







            Popular posts from this blog

            Triangular numbers and gcdProving sum of a set is $0 pmod n$ if $n$ is odd, or $fracn2 pmod n$ if $n$ is even?Is greatest common divisor of two numbers really their smallest linear combination?GCD, LCM RelationshipProve a set of nonnegative integers with greatest common divisor 1 and closed under addition has all but finite many nonnegative integers.all pairs of a and b in an equation containing gcdTriangular Numbers Modulo $k$ - Hit All Values?Understanding the Existence and Uniqueness of the GCDGCD and LCM with logical symbolsThe greatest common divisor of two positive integers less than 100 is equal to 3. Their least common multiple is twelve times one of the integers.Suppose that for all integers $x$, $x|a$ and $x|b$ if and only if $x|c$. Then $c = gcd(a,b)$Which is the gcd of 2 numbers which are multiplied and the result is 600000?

            Ingelân Ynhâld Etymology | Geografy | Skiednis | Polityk en bestjoer | Ekonomy | Demografy | Kultuer | Klimaat | Sjoch ek | Keppelings om utens | Boarnen, noaten en referinsjes Navigaasjemenuwww.gov.ukOffisjele webside fan it regear fan it Feriene KeninkrykOffisjele webside fan it Britske FerkearsburoNederlânsktalige ynformaasje fan it Britske FerkearsburoOffisjele webside fan English Heritage, de organisaasje dy't him ynset foar it behâld fan it Ingelske kultuergoedYnwennertallen fan alle Britske stêden út 'e folkstelling fan 2011Notes en References, op dizze sideEngland

            Հադիս Բովանդակություն Անվանում և նշանակություն | Դասակարգում | Աղբյուրներ | Նավարկման ցանկ